Đến nội dung

Nguyenhuyen_AG

Nguyenhuyen_AG

Đăng ký: 09-09-2010
Offline Đăng nhập: 10-01-2019 - 16:22
****-

Chứng minh $x^2+xy+y^2 \geqslant 0$

16-04-2017 - 23:08

Cho hai số thực $x,\,y$ bất kỳ. Chứng minh rằng

\[x^2 + xy + y^2 \geqslant 0.\]

Mình có hai lời giải là dùng phân tích

\[x^2 + xy + y^2 = \frac14(2x+y)^2+\frac34y^2 \geqslant 0,\]

\[x^2 + xy + y^2 =\frac{x^2+y^2+(x+y)^2}{2} \geqslant 0.\]

Bạn nào có chứng minh khác cho mình tham khảo. :)


Epsilon số 13

14-02-2017 - 13:47

Đây là số cuối cùng của tạp chí, bạn nào quan tâm có thể tải về ở đây.


$x^4 + x + \frac{1}{2} > 0$

24-01-2017 - 19:38

Cho số thực $x$ bất kỳ. Chứng minh rằng \[x^4 + x + \frac{1}{2} > 0.\]

P/s. Dạo này thấy thích bất đẳng thức một biến, đơn giản, dễ chơi, dễ trúng.


Phân tích bình phương

10-08-2016 - 16:28

Có một số bạn nhắn tin hỏi mình về cách phân tích một bất đẳng thức nào đó về dạng tổng các bình phương hôm nay mình viết một bài nhỏ chia sẻ cách làm của mình. Bài viết này mình hướng dẫn cách làm đối với bất đẳng thức bậc $3$ các bậc khác thực hiện tương tự. :D

 

Xét ví dụ đơn giản nhất

 

Ví dụ 1. Cho $a,b,c$ là ba số thực không âm. Chứng minh rằng

\[f_1 = a^3 + b^3 + c^3 - 3abc \geqslant 0.\]

Cách giải đơn giản của bài toán này là sử dụng phân tích

\begin{equation}\label{eq2}f_1 = \frac{1}{2}(a+b+c)[(a-b)^2+(b-c)^2+(c-a)^2].\end{equation}

Ta xét một dạng tổng quát của bất đẳng thức đẳng thức bậc $3$ ba biến

\[F_1 = (m_3a+m_2b+m_1c)(a-b)^2+(m_3b+m_2c+m_1a)(b-c)^2+(m_3c+m_2a+m_1b)(c-a)^2,\]

trong đó $a,b,c$ là biến và $m_i \geqslant 0,\,(i = \overline{1,3})$ là hằng số (hàm $F_1$ này sinh ra tự động bằng một hàm do mình viết nên cách đánh số hơi không giống bình thường :D).

 

Khai triển và nhóm $F_1$ lại dưới dạng

\[F_1 = (m_2+m_3)(a^3+b^3+c^3)+(m_1-2m_2+m_3)\sum ab(a+b)-6m_1abc.\]

Đồng nhất hệ số của $f$ và $F_1$ ta được

\[\left\{\begin{aligned}& m_2 + m_3 = 1 \\& m_1-2m_2+m_3 = 0\\&-6m_1 =-3\end{aligned}\right.\]

Giải hệ này ta được nghiệm $m_1 = m_2 = m_3 = \frac{1}{2}$ và dẫn đến phân tích \eqref{eq2} ở trên.

 

Ví dụ 2. Cho $a,b,c$ là ba số thực dương. Chứng minh rằng 

\[f_2 = \frac{a}{b+c} + \frac{b}{c+a} + \frac{c}{a+b} - \frac{3}{2} \geqslant 0.\]

 \[f_2 = \frac{\displaystyle 2(a^3+b^3+c^3)-ab(a+b)-bc(b+c)-ca(c+a)}{2(a+b)(b+c)(c+a)}.\]

Nên ta có hệ phương trình đồng nhất hệ số như sau

\[\left\{\begin{aligned}& m_2 + m_3 = 2 \\& m_1-2m_2+m_3 = -1\\&-6m_1 = 0\end{aligned}\right.\]

Giải hệ này ta được nghiệm $m_1 = 0, \,m_2 = 1, \,m_3 = 1$ từ đó dẫn đến

\[f_2 = \frac{(a+b)(a-b)^2+(b+c)(b-c)^2+(c+a)(c-a)^2}{2(a+b)(b+c)(c+a)}.\]

Ví dụ 3. Cho $a,b,c$ là ba số thực dương. Chứng minh rằng

\[f_3= (a+b+c)^3 - 27abc \geqslant 0.\]

\[f_3 = a^3+b^3+c^3+3\sum ab(a+b) - 21abc.\]

Nên ta có hệ phương trình đồng nhất hệ số như sau

\[\left\{\begin{aligned}& m_2 + m_3 = 1 \\& m_1-2m_2+m_3 = 3\\&-6m_1 = -21\end{aligned}\right.\]

Giải hệ này ta được $m_1 = \frac{7}{2},\,m_2 =\frac{1}{2},\,m_3 = \frac{1}{2}$ và có phân tích

\[f_3 = \frac{1}{2}(a+b+7c)(a-b)^2 + \frac{1}{2}(b+c+7a)(b-c)^2 + \frac{1}{2}(c+a+7b)(c-a)^2.\]

Hàm $F_1$ ở trên xử lý tương đối tốt đối với các bất đẳng thức đối xứng hoặc hoán vị tuy nhiên đối với các bài toán không đối xứng cũng không hoán vị thì sao, chẳng hạn như bài toán sau

 

Ví dụ 4. Cho $a,b,c$ là ba số thực dương. Chứng minh rằng

\[f_4 = \frac{a+3c}{a+b}+\frac{c+3a}{b+c}+\frac{4b}{c+a} - 6 \geqslant 0.\]

 Rõ ràng với bài toán này thì hàm $F_1$ tỏ ra không hiệu quả ta cần tìm một hàm khác tốt hơn hay nói cách khác là tổng quát hơn.

 

Ta xét

\[F_2 = (m_3a+m_2b+m_1c)(a-b)^2+(m_6a+m_5b+m_4c)(b-c)^2+(m_9a+m_8b+m_7c)(c-a)^2,\]

với $m_i \geqslant 0 \, (i = \overline{1,9}).$

 

Gọi $\text{coef}_F\,a^3$ là hệ số của $a^3$ trong $F$ ta có

\[\left\{\begin{aligned}& \text{coef}_F\;a^3 = m_3 + m_9 \\& \text{coef}_F\;a^3 = m_2 + m_5 \\& \text{coef}_F\;c^3 = m_4 + m_7 \\& \text{coef}_F\;a^2b = m_2 -2m_3 + m_8 \\& \text{coef}_F\;b^2c = m_1 + m_4 -2m_5 \\& \text{coef}_F\;c^2a = m_6 - 2m_7 + m_9 \\&\text{coef}_F\;ab^2 = -2m_2 + m_3 + m_6 \\& \text{coef}_F\;bc^2 = -2m_4 + m_5 + m_8 \\& \text{coef}_F\;ca^2 = m_1 + m_7 - 2m_9 \\& \text{coef}_F\;abc = -2m_1 - 2m_6 - 2m_8\end{aligned}\right.\]

Lại có

\[f_4 = \frac{3a^3+4b^3+3c^3-2a^2b-2b^2c-c^2a-2ab^2-2bc^2-ca^2}{(a+b)(b+c)(c+a)}.\]

Đồng nhất hệ số của $f_4$ và $F_2$ ta được

\[\left\{\begin{aligned}& m_3 + m_9 = 3\\& m_2 + m_5 = 4\\& m_4 + m_7 = 3\\& m_2 -2m_3 + m_8 = -2 \\& m_1 + m_4 -2m_5 -2 \\& m_6 - 2m_7 + m_9 = -1\\& -2m_2 + m_3 + m_6 = -2\\& -2m_4 + m_5 + m_8 - 2\\& m_1 + m_7 - 2m_9 = -1 \\& -2m_1 - 2m_6 - 2m_8 = 0\end{aligned}\right.\]

Giải hệ phương trình này ta được $m_1= 0,\,m_2= 2,\,m_3= 2,\,m_4= 2,\,m_5= 2,\,m_6= 0,\,m_7= 1,\,m_8= 0,\,m_9= 1$ từ đó dẫn đến

\[f_4 =  \frac{2(a+b)(a-b)^2+2(b+c)(b-c)^2+(c+a)(c-a)^2}{(a+b)(b+c)(c+a)}.\]

Sử dụng $F_2$ đề giải lại các ví dụ trước ta tìm được thêm một số kết quả như sau

\[f_1 = \frac{1}{2}(a+2b)(a-b)^2+\frac{3}{2}a(b-c)^2+\frac{1}{2}(2c+a)(c-a)^2,\]

\[f_3 = \frac{1}{2}(2a+2b+5c)(a-b)^2+\frac{1}{2}(c+8a)(b-c)^2+\frac{1}{2}(8b+c)(c-a)^2.\]

Trên đây là một trong số cách mà mình dùng để phân tích bình phương cho các bất đẳng thức thuần nhất tuy nhiên nó chưa phải là cách tốt nhất. Có một phương pháp tối ưu hơn đó là sử dụng thuật toán SUI của SUI Zhen-lin (thành viên szl6208 trên diễn đàn AoPS).

 

Trong lớp các bất đẳng thức bậc $3$ ba biến số không thể không nhắc đến hai bài toán nổi tiếng sau

 

Với $a,b,c$ là ba số thực không âm, khi đó

\[f_5 = 4(a+b+c)^3 - 27(a^2b+b^2c+c^2a+abc) \geqslant 0,\]

\[f_6 = a(a-b)(a-c)+b(b-c)(b-a)+c(c-a)(c-b) \geqslant 0.\]

Như đã biết $f_5$ là một bất đẳng thức nổi tiếng của GS.Vasile Cîrtoaje còn $f_6$ chính là bất đẳng thức Schur bậc $3$  quen thuộc.Và đây là phân tích của hai bài toán nổi tiếng này

\[f_5 = \displaystyle  \frac{1}{6(a+b+c)}\sum \left [18ab+(4a+b-2c)^2\right](a-2b+c)^2,\]

\[f_6 = \frac{\displaystyle \left [ \sum ab(a+b)-6abc \right ](a-b)^2(b-c)^2(c-a)^2+abc\left(\sum a^2 - \sum bc\right)^3}{(a^2b+b^2c+c^2a-3abc)(ab^2+bc^2+ca^2-3abc)}.\]

Đây là hai video demo chương trình phân tích bình phương do mình viết: Video 1, Video 2.

 

Bài viết gốc ở đây.


$(a-b)^2(b-c)^2(c-a)^2$

27-07-2016 - 22:14

Gửi tặng đến các thành viên của VMF bài viết của mình cho kỷ yếu GGTH năm nay. Ban đầu bài này nằm chung một chuyên đề với Bổ đề hoán vị nhưng đọc lại thấy nó dài quá nên mình chia nó ra làm 3 phần: Bổ đề hoán vị, $(a-b)^2(b-c)^2(c-a)^2$ và còn một phần nữa vẫn đang viết.

 

Mọi người đọc bài viết và cho tác giả ý kiến nhé. :)

 

 

$(a-b)^2(b-c)^2(c-a)^2$

 

 

Trong bài viết nhỏ này tác giả xin được giới thiệu với bạn đọc những ứng dụng thú vị xoay quanh việc khai thác đại lượng $(a-b)^2(b-c)^2(c-a)^2$ kết hợp với kỹ thuật $pqr.$ Dù những lời giải và biến đổi trong bài đôi khi khá cồng kềnh nhưng hướng đi lại rất trực quan và hiệu quả. Hy vọng kỹ thuật này sẽ cung cấp cho bạn đọc có một số ý tưởng ban đầu trong lúc bối rối trước một bất đẳng thức khó, cũng như từ việc đánh giá và đẳng thức sẽ tìm được lời giải gọn và đẹp hơn.

 

Với mọi số thực $a,\,b,\,c$ bất kỳ ta luôn có
\begin{equation} \label{lab-1} % :v
P = (a-b)^2(b-c)^2(c-a)^2 \geqslant 0.
\end{equation}
Nếu đặt $p = a+b+c, \, q = ab+bc+ca$ và $r = abc$ (cách đặt này sẽ thống nhất cho cả bài viết) thì ta có thể biểu diễn
\begin{equation} \label{lab0}
P = p^2q^2 - 4q^3 + 2p(9q - 2p^2)r - 27r^2 = \frac{4(p^2-3q)^3-(2p^3-9pq+27r)^2}{27}.
\end{equation}
Đối với những ai từng nghiên cứu và sử dụng qua kỹ thuật $pqr,$ khai triển trên không quá xa lạ nhưng lại ít được sử dụng vì khá cồng kềnh. Bài viết này sẽ khai thác trực tiếp biến đổi trên và ứng dụng trong xử lý một số biểu thức hoán vị. Hy vọng qua đây, bạn đọc sẽ tìm thấy sự thú vị riêng cũng như nét độc đáo của bất đẳng thức.

 

1. Các bài toán mở đầu

Trong phần này, ta sẽ dùng kỹ thuật $pqr$ để trực tiếp xử lý những bất đẳng thức có chứa đại lượng $(a-b)(b-c)(c-a).$ Trước tiên, dùng khai triển \eqref{lab0} để chuyển bài toán về dạng $pqr.$ Sau đó, dựa trên những điều kiện của đề bài tìm ra mối liên hệ giữa $p,\,q$ và $r.$ Từ đó, có những đánh giá thích hợp hoặc dồn về một biến và khảo sát hàm. Để hiểu rõ hơn, mời bạn đọc cùng xem các bài toán sau đây.

Bài 1. Cho ba số thực $a,\,b,\,c$ và $t \geqslant 0$ cho trước thỏa mãn điều kiện $a+b+c=0,$ $a^2+b^2+c^2 = 6t^2.$ Tìm giá trị lớn nhất của biểu thức
\begin{equation} \label{lab1}
F = \left|(a-b)(b-c)(c-a)\right|.
\end{equation}

(Nguyễn Văn Huyện)

Lời giải. Từ giả thiết ta có
\[q = \frac{(a+b+c)^2 - (a^2+b^2+c^2)}{2} = -3t^2.\]
do đó $p = 0,\;q = -3t^2,$ suy ra
\[\begin{aligned}F^2 & = p^2q^2 - 4q^3 + 2p(9q - 2p^2)r - 27r^2 \\ & = 27(4t^6 - r^2) \leqslant 27 \cdot 4t^6 = 108t^6.\end{aligned}\]
Vậy $F \leqslant 6\sqrt{3}t^3$ đẳng thức xảy ra khi và chỉ khi
\[\left\{ \begin{aligned} & abc = 0 \\& a + b + c = 0 \\& ab + bc + ca = -3t^2 \end{aligned} \right.\]
Tức $a,\,b,\,c$ lần lượt là ba nghiệm của
\[x^3 - 3t^2x = 0.\]
Giải phương trình này ta được $(a, \, b, \, c) = (\sqrt{3}t, \, -\sqrt{3}t ,\, 0)$ cùng các hoán vị. Điều này cho phép ta kết luận $F_{\max} = 6\sqrt{3}t^3.$

Nhận xét. Trường hợp $t = 1$ ta được bài toán sau:

Cho ba số thực $a,\,b,\,c$ thỏa mãn $a+b+c=0$ và $a^2+b^2+c^2 = 6.$ Tìm giá trị lớn nhất của biểu thức
\[F = \left|(a-b)(b-c)(c-a)\right|.\]

(Turkey JBMO 2014)

Bài 2. Cho $a, \, b, \, c$ là ba số thực bất kỳ. Chứng minh rằng
\[{{(a-b)}^{3}}{{(b-c)}^{3}}+{{(b-c)}^{3}}{{(c-a)}^{3}}+{{(c-a)}^{3}}{{(a-b)}^{3}}+\frac{15}{4}{{(a-b)}^{2}}{{(b-c)}^{2}}{{(c-a)}^{2}} \leqslant 0.\]

(Liu Qian Bao, Nguyễn Văn Huyện)

Lời giải. Đặt $x=a-b,\,\,y=b-c,\,\,z=c-a$ thì $x+y+z=0$ bài toán trở thành
\begin{equation} \label{lab3}
{{x}^{3}}{{y}^{3}}+{{y}^{3}}{{z}^{3}}+{{z}^{3}}{{x}^{3}}+\frac{15}{4}{{x}^{2}}{{y}^{2}}{{z}^{2}} \leqslant 0.
\end{equation}

\[\begin{aligned} {{x}^{3}}{{y}^{3}}+{{y}^{3}}{{z}^{3}}+{{z}^{3}}{{x}^{3}} & ={{(xy+yz+zx)}^{3}}-3(xy+yz)(yz+zx)(zx+xy) \\& = {{(xy+yz+zx)}^{3}}+3{{x}^{2}}{{y}^{2}}{{z}^{2}} \\& = q^3+3r^2,\end{aligned}\]
nên \eqref{lab3} tương đương với
\[4q^3+27r^2 \leqslant 0.\]
Dễ thấy đây chính là \eqref{lab0} trong trường hợp $p = 0$ nên nó hiển nhiên đúng. Đẳng thức xảy ra khi và chỉ khi $a+b=2c,\,b+c=2a$ hoặc $c+a=2b.$ Bài toán được chứng minh.

Bài 3. Tìm hằng số $M$ nhỏ nhất sao cho bất đẳng thức
\[\left | ab(a^2-b^2)+bc(b^2-c^2)+ca(c^2-a^2) \right | \leqslant M(a^2+b^2+c^2)^2,\]
luôn đúng với mọi số thực $a,\,b,\,c$ thay đổi bất kỳ.

(Finbarr Holland, IMO 2006)

Lời giải. Ta chỉ cần xét $a^2+b^2+c^2>0,$ và thấy yêu cầu của bài toán đồng nghĩa với việc tìm giá trị lớn nhất của biểu thức
\[\begin{aligned} F & = \frac{\left|ab(a^2-b^2)+bc(b^2-c^2)+ca(c^2-a^2)\right|}{(a^2+b^2+c^2)^2} \\& = \frac{\left | (a+b+c)(a-b)(b-c)(c-a)\right |}{(a^2+b^2+c^2)^2}. \end{aligned}\]
Nếu thay $(a,b,c)$ bởi $(-a,-b,-c)$ thì bài toán vẫn không đổi nên ta có thể giả sử $a+b+c>0$ và chuẩn hóa $a+b+c=1,$ khi đó
\[F = \frac{\left|(a-b)(b-c)(c-a)\right|}{(1-2q)^2} = \frac{\sqrt{\frac{4(1 - 3q)^3 - [27r - (9q-2)]^2}{27}}}{(1-2q)^2} \leqslant \frac{2(1 - 3q)\sqrt{3(1 - 3q)}}{9(1 - 2q)^2}.\]
Đặt
\[f(q) = \frac{2(1 - 3q)\sqrt{3(1 - 3q)}}{9(1 - 2q)^2},\]
thì
\[f^{'}(q) = \frac{(1+6q)(3q-1)}{3(2q-1)^3\sqrt{3(1-3q)}},\]
do đó phương trình $f^{'}(q) = 0$ có nghiệm duy nhất $q = -\frac{1}{6}.$ Lập bảng biến thiên ta thấy
\[f(q) \leqslant f\left(-\frac{1}{6}\right)= \frac{9\sqrt{2}}{32}.\]
Do đó $F \leqslant \frac{9\sqrt{2}}{32},$ đẳng thức xảy ra khi $p = 1,\,q = - \frac{1}{6},\,r = - \frac{7}{54},$ tức $a,\,b,\,c$ lần lượt là ba nghiệm của
\[t^3 - t^2 - \frac{1}{6}t + \frac{7}{54} = 0.\]
Từ đó ta được
\[a = \frac{2-3\sqrt{2}}{2} \cdot c , \; b = \frac{2+3\sqrt{2}}{2} \cdot c,\]
hoặc
\[a = \frac{-11+6\sqrt{2}}{7} \cdot c, \; b = \frac{-2+3\sqrt{2}}{7} \cdot c.\]
Vậy giá trị nhỏ nhất cần tìm là $M = \frac{9\sqrt{2}}{32}.$

Bài 4. Tìm hằng số $k$ lớn nhất sao cho bất đẳng thức
\begin{equation} \label{80}
\frac{a^2+b^2+c^2}{ab+bc+ca} - 1 \geqslant k\left ( \frac{a-b}{a+b}+\frac{b-c}{b+c}+\frac{c-a}{c+a} \right )^2,
\end{equation}
luôn đúng với mọi số thực $a,\,b,\,c$ không âm thỏa mãn $ab+bc+ca>0.$

(Nguyễn Văn Huyện)

Lời giải. Với $a = 2 + \sqrt{3},\,b=1,\,c=0$ thì \eqref{80} trở thành $k \leqslant 9.$ Ta sẽ chứng minh $k = 9$ là giá trị lớn nhất cần tìm, tức chứng minh
\[\frac{a^2+b^2+c^2}{ab+bc+ca} - 1 \geqslant 9\left ( \frac{a-b}{a+b}+\frac{b-c}{b+c}+\frac{c-a}{c+a} \right )^2,\]
hay là
\[\frac{a^2+b^2+c^2}{ab+bc+ca} - 1 \geqslant \frac{9(a-b)^2(b-c)^2(c-a)^2}{(a+b)^2(b+c)^2(c+a)^2}.\]
Đổi biến về $pqr$ như sau
\[\frac{p^2-3q}{q} \geqslant \frac{9\left[p^2q^2-4q^3+2p(9q-2p^2)r-27r^2\right]}{(pq-r)^2},\]
hoặc
\begin{equation} \label{lab8}
(p^2-3q)(pq-r)^2 \geqslant 9q\left[p^2q^2-4q^3+2p(9q-2p^2)r-27r^2\right].
\end{equation}
Chuẩn hóa $p=1$ và nhóm lại theo $r$ bất đẳng thức \eqref{lab8} trở thành
\[(1 + 240q)r^2 + 2q(17 - 78q)r + q^2(1 - 6q)^2 \geqslant 0.\]
Nếu $0 \leqslant q \leqslant \frac{17}{78}$ thì bất đẳng thức hiển nhiên đúng. Còn nếu $\frac{17}{78} \leqslant q \leqslant  \frac{1}{3}$ đặt
\[f(r) = (1 + 240q)r^2 + 2q(17 - 78q)r + q^2(1 - 6q)^2.\]
Ta có
\[\begin{aligned}\Delta_{f(r)}^{'} & = [q(17-78q)]^2-(1+240q)q^2(1-6q)^2 \\& = - 288q^2(1 - 3q)(1 - 2q)(5q - 1). \end{aligned}\]
Dễ thấy $1 \geqslant 3q \geqslant 2q$ và
\[5q - 1 \geqslant 5\cdot \frac{17}{78} - 1 = \frac{7}{78} > 0.\]
Cho nên $\Delta_{f(r)}^{'} \leqslant 0$ suy ra $f(r) \geqslant 0.$ Đẳng thức xảy ra khi $a=b=c$ hoặc $\frac{a}{b} = 2 \pm \sqrt{3} ,\,c=0$ cùng các hoán vị.

Bài 5. Với $a,\,b,\,c$ là ba số thực dương và số thực $k \geqslant 0$ cho trước thỏa mãn điều kiện
\[(a+b+c)\left ( \frac{1}{a}+\frac{1}{b}+\frac{1}{c} \right )=(k+3)^2.\]
Chứng minh rằng
\[\left |\frac{(a-b)(b-c)(c-a)}{abc}  \right | \leqslant \sqrt{k^3(k+4)}.\]

(Nguyễn Văn Huyện)

Lời giải. Chú ý rằng
\begin{equation} \label{lab7}
\begin{aligned}\left |\frac{(a-b)(b-c)(c-a)}{abc}  \right | & = \sqrt{\frac{(a-b)^2(b-c)^2(c-a)^2}{a^2b^2c^2}} \\& = \sqrt{\frac{p^2q^2 - 4q^3 + 2p(9q - 2p^2)r - 27r^2}{r^2}}.\end{aligned}
\end{equation}
Từ giả thiết ta có $\displaystyle p = \frac{r(k+3)^2}{q},$ thay giá trị này vào \eqref{lab7} và biến đổi, ta được
\[\left |\frac{(a-b)(b-c)(c-a)}{abc}  \right | = \sqrt{(k+3)^4+18(k+3)^2-27-4\left[\frac{(k+3)^6r^2}{q^3}+\frac{q^3}{r^2} \right]}.\]
Theo bất đẳng thức AM-GM thì
\[\frac{(k+3)^6r^2}{q^3}+\frac{q^3}{r^2} \geqslant 2(k+3)^3,\]
do đó
\[\left |\frac{(a-b)(b-c)(c-a)}{abc}  \right | \leqslant \sqrt{(k+3)^4+18(k+3)^2-27-8(k+3)^3} = \sqrt{k^3(k+4)}.\]
Đẳng thức xảy ra khi chẳng hạn
\[\frac{a}{c} = \frac{k+2+\sqrt{k(k+4)}}{2},\,\frac{b}{c} = \frac{k+2-\sqrt{k(k+4)}}{2}.\]
Chứng minh hoàn tất.

Bài 6. Chứng minh rằng
\begin{equation} \label{lab5}
(x^2-x+1)(y^2-y+1)(z^2-z+1) \geqslant (x-y)(y-z)(z-x),
\end{equation}
trong đó $x,\,y,\,z$ là ba số thực thay đổi bất kỳ.

(Ji Mun Kwon)

Lời giải. Thay $(x,y,z)$ bởi $\left ( \frac{1+a}{2},\frac{1+b}{2},\frac{1+c}{2} \right )$ bất đẳng thức \eqref{lab5} trở thành
\[(a^2+3)(b^2+3)(c^2+3) \geqslant 8(a-b)(b-c)(c-a).\]
Theo bất đẳng thức AM-GM thì
\[\frac{q^2}{3} + 6q + 27 \geqslant 0,\]

\[p^2 + 2pr + r^2 \geqslant 0.\]
Do đó
\[\begin{aligned}(a^2+3)(b^2+3)(c^2+3) & =a^2b^2c^2+27+3(a^2b^2+b^2c^2+c^2a^2)+9(a^2+b^2+c^2) \\&= r^2+27+3(3p^2+q^2-2pr-6q) \\ & \geqslant -(p^2+2pr) -\left(\frac{q^2}{3} + 6q\right)+ 3(3p^2+q^2-2pr-6q) \\ &=\frac{8}{3}\left[3(p^2-3q) + (q^2-3pr) \right]\\ & \geqslant \frac{16}{3} \sqrt{3(p^2-3q)(q^2-3pr)}.\end{aligned}\]
Như vậy ta cần chỉ ra
\[4(p^2-3q)(q^2-3pr) \geqslant 3(a-b)^2(b-c)^2(c-a)^2,\]
tương đương với
\[4(p^2-3q)(q^2-3pr) \geqslant 3\big[p^2q^2-4q^3+2p(9q-2p^2)r-27r^2\big],\]
thu gọn thành
\[(pq-9r)^2 \geqslant 0.\]
Đẳng thức xảy ra khi và chỉ khi $y = \frac{z-1}{z},\,x = \frac{1}{1-z},\,z \ne 0,\,z \ne 1$ cùng các hoán vị. Chứng minh hoàn tất.

Nhận xét. Lời giải dựa vào ý tưởng của anh Võ Quốc Bá Cẩn trên diễn đàn AoPS.

Bài 7. Cho ba số thực $a,\,b,\,c$ thỏa mãn đồng thời các điều kiện
\[a+b+c=0, \; a^2+b^2+c^2=3.\]
Chứng minh rằng
\begin{equation} \label{41}
-\frac{9}{2} \leqslant a^5b+b^5c+c^5a \leqslant -3.
\end{equation}

(Nguyễn Văn Huyện, Lê Việt Hải)

Lời giải. Từ giả thiết ta có
\[\left\{\begin{aligned} & ab+bc+ca = - \frac{3}{2} \\& a^2b^2+b^2c^2+c^2a^2 = \frac{9}{4} \\ & a^3+b^3+c^3 = 3abc \end{aligned}\right.\]

\[\begin{aligned}a^5b+b^5c+c^5a & = (a+b+c) \sum a^4b - \left(a^4b^2+b^4c^2+c^4a^2 + abc \sum a^3\right) \\& = -(a^4b^2+b^4c^2+c^4a^2+3a^2b^2c^2).\end{aligned}\]
nên ta có thể viết \eqref{41} lại dưới dạng
\begin{equation} \label{lab4}
3 \leqslant a^4b^2+b^4c^2+c^4a^2+3a^2b^2c^2 \leqslant \frac{9}{2}.
\end{equation}
Lại có
\[\begin{aligned} 2(a^4b^2+b^4c^2+c^4a^2) & = \sum a^2 \sum b^2c^2 - 3a^2b^2c^2 - \prod (a^2-b^2) \\ & =\frac{27}{4} - 3a^2b^2c^2 - \prod (a^2-b^2),\end{aligned}\]
nên \eqref{lab4} tương đương với
\[3a^2b^2c^2 - \frac{9}{4} \leqslant (a^2 - b^2)(b^2 - c^2)(c^2 - a^2) \leqslant 3a^2b^2c^2 + \frac{3}{4}.\]
hay
\[3r^2 - \frac{9}{4} \leqslant (a^2 - b^2)(b^2 - c^2)(c^2 - a^2) \leqslant 3r^2 + \frac{3}{4}.\]
Ta chứng minh
\[(a^2 - b^2)(b^2 - c^2)(c^2 - a^2) \leqslant 3r^2 + \frac{3}{4}.\]
Thật vậy, vì
\[\begin{aligned}(a^2 - b^2)(b^2 - c^2)(c^2 - a^2) & \leqslant \left|(a^2-b^2)(b^2-c^2)(c^2-a^2)\right| \\& = \sqrt{(a+b)^2(b+c)^2(c+a)^2\cdot(a-b)^2(b-c)^2(c-a)^2}  \\ & = \sqrt{(pq-r)^2\left[p^2q^2-4q^3+2p(9q-2p^2)r-27r^2\right]} \\& = \sqrt{\frac{27r^2(1 - 2r^2)}{2}},\end{aligned}\]

\[\frac{27r^2(1 - 2r^2)}{2} - \left(3r^2 + \frac{3}{4}\right)^2 = -\frac{9}{16}(8r^2 - 1)^2 \leqslant  0,\]
nên bất đẳng thức trên đúng. Đẳng thức xảy ra khi và chỉ khi $r^2 = \frac{1}{8},\,p = 0$ và $q = -\frac{3}{2}$ tức $a, \, b, \, c$ là ba nghiệm của
\[\left\{ \begin{aligned} & \left[ \begin{aligned} & {{t}^{3}}-\frac{3}{2}t-\frac{1}{\sqrt{8}}=0 \\& {{t}^{3}}-\frac{3}{2}t+\frac{1}{\sqrt{8}}=0\end{aligned} \right. \\& ({{a}^{2}}-{{b}^{2}})({{b}^{2}}-{{c}^{2}})({{c}^{2}}-{{a}^{2}}) \geqslant 0
\end{aligned} \right.\]
Giải hệ phương trình này ta được $(a, \, b, \, c)$ là một hoán vị của một trong hai bộ ba dưới đây
\[\left\{\left(\sqrt{2}\cos \frac{4\pi}{9},\,\sqrt{2}\cos \frac{2\pi}{9},\,-\sqrt{2}\cos \frac{\pi}{9}\right),\,\left(-\sqrt{2}\cos \frac{4\pi}{9},\,-\sqrt{2}\cos \frac{2\pi}{9},\,\sqrt{2}\cos \frac{\pi}{9}\right)\right\}.\]
Tiếp đến ta chứng minh
\[(a^2 - b^2)(b^2 - c^2)(c^2 - a^2) \geqslant - \frac{9}{4} + 3r^2.\]
Đánh giá tương tự, ta có
\[(a^2 - b^2)(b^2 - c^2)(c^2 - a^2) \geqslant - \sqrt{\frac{27r^2(1 - 2r^2)}{2}}.\]
Do đó ta chỉ cần chỉ ra
\[\sqrt{\frac{27r^2(1 - 2r^2)}{2}} \leqslant \frac{9}{4} - 3r^2.\]
Chú ý rằng $r^2 \leqslant \frac{1}{2}$ cho nên bất đẳng thức này đúng vì
\[\frac{27r^2(1 - 2r^2)}{2} - \left (\frac{9}{4} - 3r^2  \right )^2 = - \frac{9}{16}(8r^2-3)^2 \leqslant 0.\]
Đẳng thức xảy ra khi và chỉ khi $r^2 = \frac{3}{8},\,p = 0$ và $q = -\frac{3}{2}$ tức $a, \, b, \, c$ là ba nghiệm của
\[\left\{ \begin{aligned} & \left[ \begin{aligned} & {{t}^{3}}-\frac{3}{2}t-\sqrt{\frac{3}{8}}=0 \\& {{t}^{3}}-\frac{3}{2}t+\sqrt{\frac{3}{8}}=0\end{aligned} \right. \\& ({{a}^{2}}-{{b}^{2}})({{b}^{2}}-{{c}^{2}})({{c}^{2}}-{{a}^{2}}) \leqslant 0
\end{aligned} \right.\]
Giải hệ phương trình này ta được $(a, \, b, \, c)$ là một hoán vị của một trong hai bộ ba dưới đây
\[\left\{\left (\sqrt{2}\cos \frac{\pi}{18},\,-\sqrt{2}\sin \frac{2\pi}{9},\,-\sqrt{2}\sin \frac{\pi}{9}\right ),\,\left (-\sqrt{2}\cos \frac{\pi}{18},\,\sqrt{2}\sin \frac{2\pi}{9},\,\sqrt{2}\sin \frac{\pi}{9}\right )\right\}.\]
Bài toán được chứng minh.